You are on page 1of 3

1.

Let 49 students solve 3 problems and the score of each problem is one of these nonnegative integers
0, 1, 2, 3, 4, 5, 6, 7. Prove that there exist two students A and B such that for each of the
problems the scores of A is not less than B. (The Problem Prepared for 29th IMO in 1988).
Sol. Proof: Suppose that there are two students A and B such that for first and second problem, their
scores are the same. If the score of the third problem A get is not less than B’s. then for each of the
problems the scores of A is not less than B's.
Now, suppose that for any two students, their scores of first and second problems are not all the
same and we represent each of the 49 students by a point (i, j) (0  i , j  7) in a plane where i, j are
his (or her) scores of the first and second problems respectively. Thus the 49 students correspond to
49 distinct integer points in plane. Set
S = {(i, j) | i, j are integer numbers and 0  i, j  7} ;
M1 = {(i, j) | (i, j)  S, 0  i  7, j = 0 or i = 7, 1  j  7};
M2 = {(i, j) | (i, j)  S, 0  i  6, j = 1 or i = 6, 2  j  7};
M3 = {(i, j) | (i, j)  S, 0  i  5, j = 2 or i = 5, 3  j  7};
M4 = {(i, j) | (i, j)  S, 0  i  4, j = 3 or i = 4, 4  j 7};
M5 = {(i, j) | (i, j)  S, i = 2, 3 and 4  j  7};
M6 = {(i, j) | (i, j)  S, i = 0, 1 and 4  j  7};
Thus the 49 integer points belong to the set
S = M1  M2  M3  M4  M5  M6
 49 − 1 
By Pigeonhole Principle, there are at least  + 1 = 9 integral points all of which belong to the
 6 
same set, denoted by M. Since | M5 | = | M6 | = 8. where |Mi| denote the numbers of elements in Mi,
M is one of M1, M2, M3 or M4. Since the 9 integral points in M correspond 9 distinct students whose
score of the third problem are 0, 1, 2, 3, 4, 5, 6, 7, there are at least two students whose scores of the
third problem are the same. By the constructions of M1, M2, M3 or M4, we know that there exists one
(denoted by A) of the two students, such that for the first and second problems, the scores of A
are not less than another student's (denoted by B). Therefore, for each problem, the scores of A are
not less than B's.
2. In a group of 17 scientists each scientist sends letters to the others. In their letters only three topics
are involved and each couple of scientists makes reference to only one topic. Show that there exists
a group of three scientists which send each other letters on the same topic. (6th IMO)
Sol. Let 17 points Ai (i = 1, 2,..., 17) in a circle represent 17 scientists. If a couple of scientists makes
reference to the first (second or third) topic, then the line segment connecting two corresponding
points is colored red (blue or green). By Pigeonhole Principle, of 16 line segments meeting A i at
16 − 1 
least  + 1 = 6 is colored with the same color. Without loss of generality, suppose that the line
 3 
segments A1A2, A1A3, .... A1A7 are colored red. If among six points A2, A3, ….. A7. there are two
points Ai , Aj(2  i < j  7) such that the line segment connecting the two points is colored red, then
there is a red triangle A1AiAj. Otherwise, the line segments connecting each pair of six points A2,
A3... A7 are all colored with blue or green, by the Ramsey's theorem, there exists a monochromatic
triangle, i.e. there exists a group of three scientists whose letters are about the same topic.
3. Is it possible to choose (1) 4; (2) 5 distinct positive integers such that the sum of any three numbers
of them is a prime?
Sol. (1 ) Numbers 1, 3, 7, and 9 satisfy the conditions of the problem. (2) Consider the remainders of any
5 positive integers with modulus 3. If three remainders 0, 1, and 2 all appear, then the sum of the
three numbers is divisible by 3, so it is not a prime. If at more two of the three remainders 0, 1, and 2
 5 − 1
occur, then by Pigeonhole Principle, at least  + 1 = 3 numbers whose remainders with modulus
 2 
3 are equal, hence the sum of the three numbers is divisible by 3, and it is not a prime. Summing up
the above, there are not five numbers such that the sum of any three of these numbers is a prime.
4. Show that n7 = n mod 42.
Sol. ap = a mod p, n7 = n mod 7 [ 7 is prime number]
le., n7 – n is divisible by 7 …(i)
Again. n7 – n = n(n6 – 1)
= n(n3 – 1)(n3 + l)
= n(n – l)(n2 + n + l)(n + l)(n2 – n + 1)
= (n – 1) n (n + 1)(n2 + n + 1)(n2 – n + 1)
Now, (n – 1)n(n + 1) being the product of three consecutive integers is divisible by 3! = 6 and hence
n7 – n = (n – 1)n(n + l)(n2 + n + l)(n2 – n + 1) is divisible by 6 ...(ii)
From Eqs. (1) and (ii), n7 – n is divisible by 42(= 6  7)
and (6, 7) = 1
(We know that if a|c, b|c and (a, b) = 1, then ab|c)
5. If m is a prime number and a, b, are two numbers less than m, prove that
am-2 + am-3b + am-4b2 +….+ bm-2 is a multiple of m.
Sol. If m is prime and a, b are two numbers both less than m.
(a, m) = 1 and (b, m) = 1
[ A prime number is coprime to every number less than it]
 By Fermat theorem.
am-1 = 1 mod m …(i)
bm-1 = l mod m …(ii)
Subtracting Eq. (ii) from Eq. (i) we get
am-1 – bm-1 = 0 mod m
 m|(am-1 – bm-1)
or m|(a – b)(am-2 + am-3b + am-4b2 +….+bm-2)
But (m, a – b)= 1 [ a < m, b < m  a – b < m and m is prime]
m|(am-2 + am-3b +….+bm-2) [ if a |bc and (a, b) = 1, then a|c]
 (am-2 + am-3b +….+ bm-2) is a multiple of m.

You might also like